Last visit was: 24 Apr 2024, 08:43 It is currently 24 Apr 2024, 08:43

Close
GMAT Club Daily Prep
Thank you for using the timer - this advanced tool can estimate your performance and suggest more practice questions. We have subscribed you to Daily Prep Questions via email.

Customized
for You

we will pick new questions that match your level based on your Timer History

Track
Your Progress

every week, we’ll send you an estimated GMAT score based on your performance

Practice
Pays

we will pick new questions that match your level based on your Timer History
Not interested in getting valuable practice questions and articles delivered to your email? No problem, unsubscribe here.
Close
Request Expert Reply
Confirm Cancel
SORT BY:
Date
Tags:
Show Tags
Hide Tags
Math Expert
Joined: 02 Sep 2009
Posts: 92902
Own Kudos [?]: 618758 [1]
Given Kudos: 81587
Send PM
avatar
Manager
Manager
Joined: 10 Aug 2015
Posts: 95
Own Kudos [?]: 240 [0]
Given Kudos: 20
Send PM
Verbal Forum Moderator
Joined: 08 Dec 2013
Status:Greatness begins beyond your comfort zone
Posts: 2101
Own Kudos [?]: 8808 [0]
Given Kudos: 171
Location: India
Concentration: General Management, Strategy
GPA: 3.2
WE:Information Technology (Consulting)
Send PM
User avatar
Manager
Manager
Joined: 21 Jul 2013
Posts: 98
Own Kudos [?]: 72 [0]
Given Kudos: 142
WE:Securities Sales and Trading (Commercial Banking)
Send PM
Re: Of the votes cast on a certain proposal, 80 more were in favor of the [#permalink]
Let x be the total of votes cast, the solution we want.
Number of people against the proposal: \(\frac{40}{100}\) x
Number of people in favor: 1 - \(\frac{40}{100}\) x = \(\frac{60}{100}\) x

Since,
Bunuel wrote:
80 more were in favor of the proposal than were against it

that means 80 is the difference of the two percentages.
\(\frac{60}{100}\) x - \(\frac{40}{100}\) x = \(\frac{20}{100}\) x

\(\frac{20}{100}\) x = 80
x = 400
Director
Director
Joined: 20 Feb 2015
Posts: 631
Own Kudos [?]: 711 [0]
Given Kudos: 74
Concentration: Strategy, General Management
Send PM
Re: Of the votes cast on a certain proposal, 80 more were in favor of the [#permalink]
favour of proposal =x+80
against the proposal=x
total votes =2x+80
now,
.4(2x+80)=x
.8x+32=x
.2x=320
x=320/2=160=votes against
total votes =320+80=400
GMAT Club Legend
GMAT Club Legend
Joined: 19 Dec 2014
Status:GMAT Assassin/Co-Founder
Affiliations: EMPOWERgmat
Posts: 21846
Own Kudos [?]: 11665 [0]
Given Kudos: 450
Location: United States (CA)
GMAT 1: 800 Q51 V49
GRE 1: Q170 V170
Send PM
Re: Of the votes cast on a certain proposal, 80 more were in favor of the [#permalink]
Expert Reply
Hi All,

This question can be solved by TESTing THE ANSWERS.

Under normal circumstances, we would start by TESTing Answer B or D first. Since B is the 'easier' value, we'll start there...

Answer B: 400 total votes

We're told that 40% of the total votes were votes AGAINST the proposal:
(.4)(400) = 160 votes against

So the remaining votes were in FAVOR of the proposal:
400 - 160 = 240 votes for

With these two numbers, the difference is 240-160 = 80, which is a MATCH for what we were told. Thus, this MUST be the answer.

Final Answer:

GMAT assassins aren't born, they're made,
Rich
Senior Manager
Senior Manager
Joined: 23 Jan 2013
Posts: 429
Own Kudos [?]: 263 [0]
Given Kudos: 43
Schools: Cambridge'16
Re: Of the votes cast on a certain proposal, 80 more were in favor of the [#permalink]
Against = x
For = x+80

get proportion

x/x+x+80=4/10

10x=8x+320 => x=160, so total number = 160*2+80=400

B
RC & DI Moderator
Joined: 02 Aug 2009
Status:Math and DI Expert
Posts: 11169
Own Kudos [?]: 31886 [1]
Given Kudos: 290
Send PM
Re: Of the votes cast on a certain proposal, 80 more were in favor of the [#permalink]
1
Kudos
Expert Reply
Bunuel wrote:
Of the votes cast on a certain proposal, 80 more were in favor of the proposal than were against it. If the number of votes against the proposal was 40 percent of the total vote, what was the total number of votes cast? (Each vote cast was either in favor of the proposal or against it.)

(A) 480
(B) 400
(C) 300
(D) 240
(E) 160


Hi,
Although a relatively simpler Question, fast thinking can save time for slightly complex Qs later..
IF votes against are 40 %, votes for will be 60%.
the difference is 20% which is given as 80..
so total will be 80/20 * 100=400
B
Tutor
Joined: 12 Oct 2010
Status:GMATH founder
Posts: 893
Own Kudos [?]: 1353 [0]
Given Kudos: 56
Send PM
Re: Of the votes cast on a certain proposal, 80 more were in favor of the [#permalink]
Expert Reply
Bunuel wrote:
Of the votes cast on a certain proposal, 80 more were in favor of the proposal than were against it. If the number of votes against the proposal was 40 percent of the total vote, what was the total number of votes cast? (Each vote cast was either in favor of the proposal or against it.)

(A) 480
(B) 400
(C) 300
(D) 240
(E) 160

\(? = t\left( {{\rm{total}}} \right)\,\,\,\,\left\{ \matrix{\\
\,{\rm{against}}\,\,\,{\rm{ = }}\,\,a\,\,\mathop = \limits^{{\rm{stem}}} \,\,{2 \over 5}t \hfill \cr \\
\,{\rm{for}}\,\,{\rm{ = }}\,\,t - a\,\,\mathop = \limits^{{\rm{stem}}} \,\,80 + a\,\,\, \Rightarrow 2a = t - 80 \hfill \cr} \right.\,\,\,\, \Rightarrow \,\,\,2\left( {{2 \over 5}t} \right) = t - 80\,\,\,\, \Rightarrow \,\,\,{1 \over 5}t = 80\,\,\,\, \Rightarrow \,\,\,\left( {\rm{B}} \right)\)


We follow the notations and rationale taught in the GMATH method.

Regards,
Fabio.
Target Test Prep Representative
Joined: 14 Oct 2015
Status:Founder & CEO
Affiliations: Target Test Prep
Posts: 18754
Own Kudos [?]: 22044 [0]
Given Kudos: 283
Location: United States (CA)
Send PM
Re: Of the votes cast on a certain proposal, 80 more were in favor of the [#permalink]
Expert Reply
Bunuel wrote:
Of the votes cast on a certain proposal, 80 more were in favor of the proposal than were against it. If the number of votes against the proposal was 40 percent of the total vote, what was the total number of votes cast? (Each vote cast was either in favor of the proposal or against it.)

(A) 480
(B) 400
(C) 300
(D) 240
(E) 160


We can let n = the total number of votes. Thus, 0.4n votes were against the proposal, and 0.6n votes were in favor of the proposal. We can create the equation:

0.6n - 0.4n = 80

0.2n = 80

n = 80/0.2 = 800/2 = 400

Answer: B
User avatar
Non-Human User
Joined: 09 Sep 2013
Posts: 32649
Own Kudos [?]: 821 [0]
Given Kudos: 0
Send PM
Re: Of the votes cast on a certain proposal, 80 more were in [#permalink]
Hello from the GMAT Club BumpBot!

Thanks to another GMAT Club member, I have just discovered this valuable topic, yet it had no discussion for over a year. I am now bumping it up - doing my job. I think you may find it valuable (esp those replies with Kudos).

Want to see all other topics I dig out? Follow me (click follow button on profile). You will receive a summary of all topics I bump in your profile area as well as via email.
GMAT Club Bot
Re: Of the votes cast on a certain proposal, 80 more were in [#permalink]
Moderators:
Math Expert
92902 posts
Senior Moderator - Masters Forum
3137 posts

Powered by phpBB © phpBB Group | Emoji artwork provided by EmojiOne